Đến nội dung

kainguyen nội dung

Có 101 mục bởi kainguyen (Tìm giới hạn từ 20-04-2020)



Sắp theo                Sắp xếp  

#390791 Cho mình hỏi định hướng học toán casio căn bản trong thời gian ngắn khoảng và...

Đã gửi bởi kainguyen on 27-01-2013 - 16:49 trong Kinh nghiệm học toán

Cho mình hỏi định hướng học toán casio căn bản trong thời gian ngắn khoảng vài tuần với. Trường mình có công văn gấp gửi về bảo mấy đứa đi thi Casio Thành Phố ở Hà Nội nhưng bọn mình chưa bao giờ tiếp xúc hay làm mấy bài kiểu Casio. Mong các bạn có kinh nghiệm chia sẻ 1 chút giúp mình. Bởi ở trường mình thầy cô giáo dạy cũng không có, tài liệu thì có down trên mạng nhưng mình cần định hướng, phương pháp. Cảm ơn các bạn rất nhiều. Hình đã gửi



#349707 cho dãy số sau 1;3;6;10;15;...;$\frac{n(n+1)}{2...

Đã gửi bởi kainguyen on 25-08-2012 - 22:32 trong Đại số

cho dãy số sau 1;3;6;10;15;...; $\frac{n(n+1)}{2}$ ;...
Cm : tổng hai số hạng liên tiếp của dãy luôn là số chính phương



Bài này đơn giản mà.

Ta có tổng 2 số hạng liên tiếp:

$A=\frac{n(n+1)}{2}+\frac{(n+1)(n+2)}{2}=(n+1)^2$

Do đó A là số chính phương.



#349706 Tính tổng $u_{2011}^{2}+v_{2011}^{2...

Đã gửi bởi kainguyen on 25-08-2012 - 22:26 trong Dãy số - Giới hạn

Cho hai dãy số dương $(u_n)$, $(v_n)$ xác đinh bởi công thức:
$\left\{\begin{matrix} u_1=v_1=\frac{\sqrt{2}}{2} \\ u_{n+1}=\frac{u_n}{4v_{n+1}^{2}-1}, v_{n+1}=\frac{v_n}{4u_{n+1}^{2}-1},n=1,2,3,... \end{matrix}\right.$
a) Tính $u_{2011}^{2}+v_{2011}^{2}$.
b) Tính $\lim{u_{n}}$ và $v_{n}$



a) Ta sẽ chứng minh quy nạp: $u_n^2+v_n^2=1$ (1)

Với $n=1$ ta có (1) đúng.

Gs (1) đúng với $n=k$ thì $u_k^2+v_k^2=1$

Thay vào công thức xác định dãy, ta có:

$[u_{k+1}(4v_{k+1}^2-1)]^2+[v_{k+1}(4u_{k+1}^2-1)]^2=1$

$\Leftrightarrow (u_{k+1}^2+v_{k+1}^2-1)(16u_{k+1}^2v_{k+1}^2+1)=0$

Từ đây suy ra (1) đúng với $n=k+1$

Vậy (1) đúng, suy ra đpcm.

b) Đề câu này bạn có chép lỗi công thức xđ dãy ko vậy?

Mình thử số công thức xđ dãy thấy đây là dãy hằng @@



#340518 $\frac{(a+b+c)^{2}}{a^{2}+b^...

Đã gửi bởi kainguyen on 26-07-2012 - 17:07 trong Bất đẳng thức và cực trị

Bài toán:Cho $a,b,c\geq 0$.Chứng minh rằng:
$\frac{(a+b+c)^{2}}{a^{2}+b^{2}+c^{2}}\geq \frac{9abc}{a^{2}b+b^{2}c+c^{2}a}$



Dễ thấy có nhiều nhất 1 trong 3 số $a,b,c$ bằng 0. Khi đó bđt hiển nhiên đúng.

Giả sử 3 số $a,b,c$ khác 0.

Khi đó, theo Cauchy - Schwarz và bđt $3(x^2+y^2+z^2) \ge (x+y+z)^2$ ta có:

$\frac{9abc}{a^{2}b+b^{2}c+c^{2}a}\leq \frac{9abc}{\sqrt{(a^2+b^2+c^2)(a^2b^2+b^2c^2+c^2a^2)}}\leq \frac{9\sqrt{3}abc}{\sqrt{a^2+b^2+c^2}.(ab+bc+ca)}$


Ta cần cm: $\dfrac{(a+b+c)^2}{\sqrt{a^2+b^2+c^2}} \ge \dfrac{9\sqrt{3}abc}{ab+bc+ca}$

Ta chuẩn hóa $a+b+c=3$ và đặt: $\left\{\begin{array}{1}a+b+c=p=3\\ab+bc+ca=q\\abc=r \end{array}\right.$

Khi đó bđt (1) trở thành: $q^2\geq 3r^2(q^2-2q)$ đúng do: $\left\{\begin{matrix}
q\leq \frac{p^2}{3}=3\\
r\leq \frac{p^3}{27}=1
\end{matrix}\right.$

Vậy bđt được cm.

Dấu bằng xảy ra khi và chỉ khi: $a=b=c=1$.



#338814 $3cos4x-8cos^{6}x+2cos^{2}x+3=0$

Đã gửi bởi kainguyen on 22-07-2012 - 09:48 trong Phương trình, Hệ phương trình Lượng giác

Ta có:

$3cos4x-8cos^6x+2cos^2x+3=0$

$\Leftrightarrow 3(2cos^22x-1)-8.(\frac{1+cos2x}{2})^3+2.\frac{1+cos2x}{2}+3=0$

$\Leftrightarrow -cos^32x+3cos^22x-2cos2x=0$

Đến đây đơn giản rồi :)



#338812 Giải Phương trình Lượng giác: $\frac{sin^4x+cos^4x}...

Đã gửi bởi kainguyen on 22-07-2012 - 09:43 trong Phương trình, Hệ phương trình Lượng giác

Ta có:

$\frac{sin^4x+cos^4x}{5sin2x}=\frac{1}{2}cot2x-\frac{1}{8sin2x}$

$\Leftrightarrow \frac{1-\frac{1}{2}sin^22x}{5sin2x}=\frac{4cos2x-1}{8sin2x}$

$\Leftrightarrow 8-4(1-cos^22x)=20cos2x-5$

$\Leftrightarrow 4cos^22x-20cos2x+9=0$



#338806 Tuyển tập một số bài phương trình, hệ phương trình thi HSG tỉnh

Đã gửi bởi kainguyen on 22-07-2012 - 09:31 trong Phương trình - hệ phương trình - bất phương trình

Làm vài bài nữa nhỉ :D

Bài 69. Giải hệ phương trình

$$\begin{cases}
x^3+x^2(2-y)+x-y(2x+1)=0 \\ 2x^2+xy-5=0
\end{cases}$$

Đề thi HSG Lạng Sơn Lớp 11 - 2011/2012

Bài 70. Giải phương trình

$$x+\frac{3x}{\sqrt{x^2+1}}=1$$

Đề thi HSG Long An Lớp 12 - 2011/2012

---------------------------

Còn bài 67,59 nữa nhé :D



Bài 69:

$\begin{cases}
x^3+x^2(2-y)+x-y(2x+1)=0 \\ 2x^2+xy-5=0
\end{cases}$

$\Leftrightarrow \left\{\begin{matrix}
x^3+2x^2+x=y(x^2+2x+1)\\
2x^2+xy-5=0
\end{matrix}\right.$

$\Leftrightarrow \left\{\begin{matrix}
(x+1)^2(x-y)=0\\
2x^2+xy-5=0
\end{matrix}\right.$

$\Leftrightarrow \begin{bmatrix}
x=-1;y=-3\\
x=y=\pm \frac{\sqrt{15}}{3}
\end{bmatrix}$

Bài 70:

$x+\frac{3x}{\sqrt{x^2+1}}=1$

$\Leftrightarrow \frac{3x}{\sqrt{x^2+1}}=1-x$

$\Leftrightarrow 3x=(1-x)\sqrt{x^2+1}$

$\Rightarrow 9x^2=(x^2-2x+1)(x^2+1)$

$\Leftrightarrow x^4-2x^3-7x^2-2x+1=0$

Với $x=0$ dễ thấy không là nghiệm của pt đã cho.

Với $x \ne 0$ ta chia 2 vế pt trên cho $x^2$ được:

$x^2-2x-7-\frac{2}{x}+\frac{1}{x^2}=0$

Đặt: $x+\frac{1}{x}=t$ ta có: $t^2-2t-9=0$

Đến đây thì đơn giản rồi :D Giải ra nghiệm rồi thử lại thấy thỏa mãn hay không.

- Bài này còn 1 cách đặt đặt thế nào nữa ý, mình quên mất rồi :) cứ nông dân thế này chắc cũng được :)

PS: Tái xuất :D



#337590 Tìm Min: $x^{2}+3x+\frac{1}{x}$$\frac{x^{2}}{x^{4}+1...

Đã gửi bởi kainguyen on 19-07-2012 - 12:14 trong Bất đẳng thức và cực trị

Tìm Min:$\frac{x^{2}}{x^{4}+1}$ với x khác 0
À, bổ sung thêm là năm nay em lên lớp 9 nhé :D



Bài này chỉ có Max thôi :D (Bạn cứ thử dùng pp miền giá trị là thấy liền :D)

Tìm Max thì đơn giản là dùng Cauchy:

$A=\frac{x^{2}}{x^{4}+1}\leq \frac{x^2}{2x^2}=\frac{1}{2}$

Dấu bằng xảy ra tại chẳng hạn $x=1$

Vậy $MaxA=\frac{1}{2}$ tại chẳng hạn $x=1$.



#337532 chứng minh $ cos^2 x+cos^2 y+cos^2 z =1$

Đã gửi bởi kainguyen on 19-07-2012 - 10:36 trong Hình học không gian

mọi người giúp mình bài này
Cho hình chóp S.ABC có SA, SB, SC đôi một vuông góc với nhau. M là một điểm nằm trong miền tam giác ABC. Gọi góc giữa SM với SA, SB, SC lần lượt là x,y,z. Chứng minh hệ thức $ cos^2 x+cos^2 y+cos^2 z =1$



Cách khác:

Gọi $(P)$ là mặt phẳng đi qua M và vuông góc với DM, cắt SA, SB, SC lần lượt tại: A', B', C'.

Khi đó hình chóp S.A'B'C' là tứ diện vuông có đường cao là DM.

Theo các quan hệ vuông góc, dễ dàng chứng minh được M là trực tâm của tam giác A'B'C'.

Ta có: x=(SB'C';AB'C'); y=(SA'C';A'B'C'); z=(SA'B';A'B'C').

và: $\frac{1}{OM^2}=\frac{1}{OA'^2}+\frac{1}{OB'^2}+\frac{1}{OC'^2}$

$\Rightarrow (\frac{OM}{OA'})^2+(\frac{OM}{OB'})^2+(\frac{OM}{OC'})^2=1$

$\Leftrightarrow cos^2 x+cos^2 y+cos^2 z =1$

Từ đây ta có đpcm.



#337104 Tuyển tập một số bài phương trình, hệ phương trình thi HSG tỉnh

Đã gửi bởi kainguyen on 17-07-2012 - 22:21 trong Phương trình - hệ phương trình - bất phương trình

Không mất tính tổng quát, giả sử $x\geq y\geq z$
Ta có $2x^{3}+3x^{2}-18\geq 2y^{3}+3y^{2}-18\geq 2z^{3}+3z^{2}-18$
Hay $y^{3}+y\geq z^{3}+z\geq x^{3}+x (*)$
Nên từ $(*)$ suy ra được $y\geq z\geq x$
Mà theo giả thiết đầu $x\geq y\geq z$ nên $x=y=z$
Thay vào pt $(1)$ ta có pt bậc 3 ẩn x.
Giải ra ta được $x=y=z=2$
Mình nghĩ mọi người nên chém chậm thôi, chậm mà chắc mà. Chém một bài tới khi nào nát bét ra á. Tìm nhiều lời giải, tổng quát bài toán. Chứ không nên đăng đề nhiều thế. Mình nghĩ đề thi sẽ không ra giống lại, chủ yếu là phương pháp làm có thể giống nhau. Nên cần quan tâm tới chất lượng hơn số lượng.



Bài này bạn làm thế này là chưa chặt chẽ, mà có thể coi là sai.

1) Hệ hoán vị vòng quanh nếu bạn giả sử theo thứ tự thì phải xét 2 TH:

TH1: $x \ge y \ge z$

TH2: $x \ge z \ge y$

Mà nói chung ta chỉ có thể giả sử số nào là min, hoặc số nào là max.

2) Hàm số của vế trái đề bài $f(t)=2t^3-3t^2-18$ đâu có phải đơn điệu hoàn toàn trên $\mathbb{R}$ đâu?

- Mình nghĩ mấy bài này cần xem xét lại về phần tư duy, lập luận.

P/S: Đề tỉnh một số năm gần đây chém gần hết rồi :D, post lâu quá thì đâm ra không theo kịp thời đại, post 30/4 cũng được nhể :D



Mình thấy đề 30/4 cho kiểu lắt léo, đánh đố lắm, có lẽ chúng ta nên lựa chọn khi cho đề 30/4 vào :D



#337091 Chứng minh rằng $(x-1)(y-1)(z-1) \le 6\sqrt{3}-10$

Đã gửi bởi kainguyen on 17-07-2012 - 22:10 trong Bất đẳng thức và cực trị

Cho $x,y,z$ thực dương thỏa $x+y+z=xyz$. Chứng minh rằng $$(x-1)(y-1)(z-1) \le 6\sqrt{3}-10$$
Gabriel Dospinescu, Marian Tetiva



Do giả sử: Nếu có 2 số không lớn hơn 1 và 1 số lớn hơn 1, không giảm tổng quát, giả sử đó là $z$.

Khi đó ta có: $xy \le 1$

Ta có: $x+y=z(xy-1)<0$. Vô lý. Do đó không thể xảy ra TH này.

Khi đó, ta xét 2 TH sau:

TH1: Nếu có 1 hoặc cả 3 số không lớn hơn 1 thì hiển nhiên có đpcm.

TH2: Cả 3 số đều lớn hơn 1. Đặt: $a=x-1;b=y-1;c=z-1$

Khi đó, ta có $x+y+z=xyz$ trở thành:

$a+b+c+3=(a+1)(b+1)(c+1)$

$\Leftrightarrow 2=abc+ab+bc+ca\leq abc+3\sqrt[3]{abc}$

Từ $2\leq abc+3\sqrt[3]{abc}$ ta suy ra được: $abc\le 6\sqrt{3}-10$.

Vậy bđt được chứng minh.

Dấu bằng xảy ra khi và chỉ khi: $a=b=c=\sqrt{3}$.



#336927 Chứng minh: \[\prod {\left( {\frac{4}{{{a^2} + {b^2}}} +...

Đã gửi bởi kainguyen on 17-07-2012 - 17:37 trong Bất đẳng thức và cực trị

Cho a, b, c > 0 và $a^{2} + b^{2} + c^{2} = 3$. Chứng minh
$\left (\frac{4}{a^{2} + b^{2}} + 1 \right )\left ( \frac{4}{b^{2} + c^{2}} + 1 \right )\left ( \frac{4}{a^{2} + c^{2}} + 1 \right ) \geq 3\left ( a + b + c \right )$



Mình nghĩ đề là:



Cho a, b, c > 0 và $a^{2} + b^{2} + c^{2} = 3$. Chứng minh
$\left (\frac{4}{a^{2} + b^{2}} + 1 \right )\left ( \frac{4}{b^{2} + c^{2}} + 1 \right )\left ( \frac{4}{a^{2} + c^{2}} + 1 \right ) \geq 3\left ( a + b + c \right )^2$



thì dấu bằng mới xảy ra được chứ :D

Giải:

Đặt: $A=\left (\frac{4}{a^{2} + b^{2}} + 1 \right )\left ( \frac{4}{b^{2} + c^{2}} + 1 \right )\left ( \frac{4}{a^{2} + c^{2}} + 1 \right )$

Ta có:

$\frac{4}{a^2+b^2}+1=\frac{2}{a^2+b^2}+\frac{2}{a^2+b^2}+1\geq 3\sqrt[3]{\frac{4}{(a^2+b^2)^2}}$

Tương tự với các bđt khác rồi nhân theo từng vế, ta được:

$A \geq 27\sqrt[3]{\frac{4^3}{[(a^2+b^2)(b^2+c^2)(c^2+a^2)]^2} }$

Theo AM - GM, ta có: $(a^2+b^2)(b^2+c^2)(c^2+a^2)\leq (\frac{2(a^2+b^2+c^2)}{3})^3=8$

Từ đây suy ra: $A\geq 27$

Mà ta có: $3(a+b+c)^2\leq 3(\sqrt{3(a^2+b^2+c^2)})^2=27$

Nên:

$\left (\frac{4}{a^{2} + b^{2}} + 1 \right )\left ( \frac{4}{b^{2} + c^{2}} + 1 \right )\left ( \frac{4}{a^{2} + c^{2}} + 1 \right ) \geq 3\left ( a + b + c \right )^2$

Dấu bằng xảy ra khi và chỉ khi $a=b=c=1$.



#336767 Tuyển tập một số bài phương trình, hệ phương trình thi HSG tỉnh

Đã gửi bởi kainguyen on 17-07-2012 - 10:54 trong Phương trình - hệ phương trình - bất phương trình

Ngẫu nhiên sao mà bạn đặt thế này ?


Thì vào link của bạn trên thấy có bạn đặt như thế nên mình cũng đặt vậy, nhân ra biến đổi ai dè được như thế nên làm vậy thôi :D

Vào đọc lại link kia lại thấy cậu ý cũng làm giống giống mình :D Chắc cái cốt yếu là đặt được thế kia, mình cũng không rõ ý tưởng lắm :D



#336763 Tuyển tập một số bài phương trình, hệ phương trình thi HSG tỉnh

Đã gửi bởi kainguyen on 17-07-2012 - 10:50 trong Phương trình - hệ phương trình - bất phương trình

bài 40: giải hệ:

$$\left\{\begin{matrix} x^6+y^8+z^{10} \leq 1 & \\ x^{2007}+y^{2009}+z^{2011} \geq 1 & \end{matrix}\right.$$

chọn đội tuyển bình định 2010-2011



Đề của bạn bị nhầm dấu của pt thứ 2 :D Phải là như trên :D Thảo nào thấy cứ lạ lạ, rõ ràng bài này mình làm rồi mà :D

Từ $x^6+y^8+z^{10} \leq 1 \Rightarrow x;y;z \in [-1;1]$

Trừ theo từng vế 2 pt đã cho, ta được:

$x^6(1-x^{2001})+y^8(1-y^{2001})+z^{10}(1-z^{2001})\leq 0$

mà $x;y;z \in [-1;1]$ nên $x^6(1-x^{2001})+y^8(1-y^{2001})+z^{10}(1-z^{2001})\geq 0$

Từ đây suy ra hệ có nghiệm khi 2 bđt trên xảy ra dấu bằng.

Vậy hệ có nghiệm $(x;y;z)=(0;0;1)$ và các hoán vị.



#336759 Tuyển tập một số bài phương trình, hệ phương trình thi HSG tỉnh

Đã gửi bởi kainguyen on 17-07-2012 - 10:41 trong Phương trình - hệ phương trình - bất phương trình

Bài 36: Giải phương trình $$16x^6-16x^5-20x^4+20x^3+5x^2+2x-7=0$$
Đề đề nghị OLYMPIC 30/4 chuyên Lê Hồng Phong - 2008



Bài gì mang nặng tính đánh đố, đúng chất olympic 30/4 thì phải.

Ta có: $16x^6-16x^5-20x^4+20x^3+5x^2+2x-7=0$

$\Leftrightarrow (x-1)(16x^5-20x^3+5x+7)=0$

$\Leftrightarrow \begin{bmatrix}
x=1\\
16x^5-20x^3+5x+7 (1)
\end{bmatrix}$

Giả sử pt (1) có nghiệm $x \in [-1;1]$ ta đặt $x=cosa$ khi đó ta có (1) trở thành:

$cos5a+7=0$. Pt vô nghiệm.

Do đó (1) có nghiệm $x$ sao cho $|x|>1$

Ta đặt: $x=\frac{1}{2}(t+\frac{1}{t}) thì 16x^5=\frac{1}{2}(t^5+\frac{1}{t^5})+20x^3-5x$

Khi đó, (1) trở thành:

$\frac{1}{2}(t^5+\frac{1}{t^5})+7=0$

Đến đây giải pt bậc 2 với ẩn là $t^5$



#336668 Tuyển tập một số bài phương trình, hệ phương trình thi HSG tỉnh

Đã gửi bởi kainguyen on 16-07-2012 - 23:46 trong Phương trình - hệ phương trình - bất phương trình

Định phắn rồi mà lại lòi ra bài hay hay nữa :D

Bài 35. Giải hệ phương trình

$$\left\{\begin{matrix}
x^2+y^2=2xy+1 & & \\
x^5+y^3+1=0 & &
\end{matrix}\right.$$

Đề thi HSG cấp Thành Phố - Hà Nội - 2011/2012

P/S: Nhìn qua đã thấy sự chênh lệch khá rõ giữa đề của một số tỉnh lẻ và các tỉnh (TP) lớn :)



Ta có:

$\left\{\begin{matrix}
x^2+y^2=2xy+1 & & \\
x^5+y^3+1=0 & &
\end{matrix}\right.$

Xét pt thứ nhất: $x^2+y^2=2xy+1$

$\Leftrightarrow (x-y)^2=1$

$\Leftrightarrow \begin{bmatrix}
x-y=1\\
x-y=-1
\end{bmatrix}$

TH1: $x-y=1$ ta có:

$x^5+y^3+1=0$

$\Leftrightarrow x^5+(x-1)^3+1=0$

Xét với $x=0$ dễ thấy là nghiệm của pt trên.

Với $x>0$ thì $x^5+(x-1)^3+1>0$

Với $x<0$ thì $x^5+(x-1)^3+1<0$

TH2:$x-y=-1 \Rightarrow x^5+y^3+1=0$

$\Leftrightarrow (y-1)^5+y^3+1=0$

Ta cũng xét tương tự như trên.

Đến đây kết luận được nghiệm rồi :D

PS: Ý bạn là đề Hà Nội dễ hơn ah =))) Bài tp HCM kia trông cồng kềnh thật @@ Ngủ đã mai tính =)))



#336655 Chứng minh: $1^{3}+2^{3}+3^{3}+...+n^...

Đã gửi bởi kainguyen on 16-07-2012 - 23:22 trong Số học

Chứng minh: $1^{3}+2^{3}+3^{3}+...+n^{3}=(1+2+3+...+n)^{2}$ (1)



Với $n=1$ thì đẳng thức hiển nhiên đúng.

Giả sử (1) đúng với $n=k$ tức là:

$1^{3}+2^{3}+3^{3}+...+k^{3}=(1+2+3+...+k)^{2}$

Ta sẽ cm (1) đúng với $n=k+1$ tức là cm:

$1^{3}+2^{3}+3^{3}+...+k^{3}+(k+1)^3=(1+2+3+...+k+k+1)^{2}$

Thật vậy, ta có:

$1^{3}+2^{3}+3^{3}+...+k^{3}+(k+1)^3=(1+2+3+...+k+k+1)^{2}$

$\Leftrightarrow (1^{3}+2^{3}+3^{3}+...+k^{3})+(k+1)^3=(1+2+3+...+k)^2+(k+1)^2+2(1+2+3+...+k)(k+1)$

$\Leftrightarrow (k+1)^3=(k+1)^2+2(1+2+3+...+k)(k+1)$

Mà: $(k+1)^2+2(1+2+3+...+k)(k+1)=(k+1)^2+2.\frac{k(k+1)(k+1)}{2}=(k+1)^3$

Do đó (1) đúng với $n=k+1$

Theo nguyên lý quy nạp, ta có đpcm.



#336631 $(x^{2}+y^{2}+\dfrac{2xy}{x+y...

Đã gửi bởi kainguyen on 16-07-2012 - 22:43 trong Đại số

$\left\{\begin{matrix} x^{2}+y^{2}+\frac{2xy}{x+y}= 1& & \\ x^{2}+2y+3\sqrt{x+y}= \sqrt{4x+3y}+\sqrt{4x+5y} & & \end{matrix}\right.$



Ta có:

$\left\{\begin{matrix} x^{2}+y^{2}+\frac{2xy}{x+y}= 1& & \\ x^{2}+2y+3\sqrt{x+y}= \sqrt{4x+3y}+\sqrt{4x+5y} & & \end{matrix}\right.$

Xét pt thứ nhất:

$x^{2}+y^{2}+\frac{2xy}{x+y}= 1$

$\Leftrightarrow (x+y)^2-2xy+\frac{2xy}{x+y}-1=0$

$\Leftrightarrow (x+y-1)(x+y+1)-2xy(1-\frac{1}{x+y})=0$

$\Leftrightarrow (x+y-1)(x+y+1)-\frac{2xy(x+y-1)}{x+y}=0$

$\Leftrightarrow (x+y-1)(\frac{x^2+y^2+x+y}{x+y})=0 $ (1)

Do $x+y \ge 0$ nên $x^2+y^2+x+y \ge 0$

Dấu bằng xảy ra khi và chỉ khi $x=y=0$ (không thỏa mãn do $x+y \ne 0$)

Do đó (1) tương đương với $x+y-1=0$

Thế vào pt thứ 2 ta được:

$x^{2}+2y+3\sqrt{x+y}= \sqrt{4x+3y}+\sqrt{4x+5y}$

$\Leftrightarrow x^2+2(-x+1)+3=\sqrt{4x+3(-x+1)}+\sqrt{4x+5(-x+1)}$

$\Leftrightarrow x^2-2x+5=\sqrt{x+3}+\sqrt{5-x}$

Đặt $\sqrt{x+3}+\sqrt{5-x}=t$ với t không âm.

$\Rightarrow -(\frac{t^2-8}{2})^2+20=t$

$\Rightarrow t=4$

Sorry bạn vì lỗi kĩ thuật vừa nãy :D



#336619 Tuyển tập một số bài phương trình, hệ phương trình thi HSG tỉnh

Đã gửi bởi kainguyen on 16-07-2012 - 22:20 trong Phương trình - hệ phương trình - bất phương trình

Bài 30. Giải hệ phương trình

$$\left\{\begin{matrix}
x+\frac{3x-y}{x^2+y^2}=3 & & \\ y-\frac{x+3y}{x^2+y^2}=0
& &
\end{matrix}\right.$$

Đề chọn đội tuyển - Chuyên Vĩnh Phúc 10/11



Dễ thấy $x=0;y=0$ không là nghiệm của hệ đã cho.

Khi đó, ta có:

$\left\{\begin{matrix}
x+\frac{3x-y}{x^2+y^2}=3 & & \\ y-\frac{x+3y}{x^2+y^2}=0
& &
\end{matrix}\right.$

$\Leftrightarrow \left\{\begin{matrix}
xy+\frac{3xy-y^2}{x^2+y^2}=3y\\
xy-\frac{x^2+3xy}{x^2+y^2}=0
\end{matrix}\right.$

Cộng theo từng vế 2 pt trên, ta được:

$2xy-1=3y\Rightarrow x=\frac{3y+1}{2y}$

Thay vào pt (2), ta được:

$4y^4-3y^2-1=0$

$\Leftrightarrow \begin{bmatrix}
x=2;y=1\\
x=1;y=-1
\end{bmatrix}$



#336498 Tuyển tập một số bài phương trình, hệ phương trình thi HSG tỉnh

Đã gửi bởi kainguyen on 16-07-2012 - 17:32 trong Phương trình - hệ phương trình - bất phương trình

Rất gọn và đẹp :)

Bài 26. Chứng minh rằng phương trình có đúng $1$ nghiệm

$$\left (\sqrt{x+1} \right)^{2011}-2(\sqrt{x+1})^3=x^3+3x^2+3x+2$$

Đề dự bị HSG Nghệ An - 10/11 :)



Điều kiện: $x \ge -1$

$\left (\sqrt{x+1} \right)^{2011}-2(\sqrt{x+1})^3=x^3+3x^2+3x+2$

$\Leftrightarrow (\sqrt{x+1})^{2011}-2(\sqrt{x+1})^3=(\sqrt{x+1})^6+1$

$\Leftrightarrow (\sqrt{x+1})^{2011}=[(\sqrt{x+1})^3+1]^2$

Đặt:$ \sqrt{x+1}=t $ với $t \ge 0$ ta được:

$t^{2011}=(t^3+1)^2$ (1)

Dễ dàng thấy (1) không có nghiệm $0 \le t \le 1$ do $t^{2011}<1<(t^3+1)^2$

Giả sử (1) có 2 nghiệm pb là $t_1;t_2$

Khi đó ta có: $\left\{\begin{matrix}
t^{2011}_1=(t^3_1+1)^2\\
t^{2011}_2=(t^3_2+1)^2
\end{matrix}\right.$

Trừ theo từng vế ta được:

$(t_1-t_2)[t^{2010}_1+...+t^{2010}_2-(t^2_1+t_1t_2+t^2_2)(t^3_1+t^3_2+2)]=0$

Hiển nhiên $t^{2010}_1+...+t^{2010}_2>(t^2_1+t_1t_2+t^2_2)(t^3_1+t^3_2+2)$ do $t_1;t_2>1$

Do đó suy ra $t_1=t_2$ vậy điều giả sử là vô lý.

Vậy pt đ(1) có nghiệm duy nhất $t \ge 0$ suy ra pt đã cho có nghiệm duy nhất.

Thèm bài rồi đấy :D



#336483 Tuyển tập một số bài phương trình, hệ phương trình thi HSG tỉnh

Đã gửi bởi kainguyen on 16-07-2012 - 17:01 trong Phương trình - hệ phương trình - bất phương trình

Cảm ơn bạn đã tham giá topic nhoé :). Bài làm trên của bạn rất tốt rồi, rõ ràng, đẹp :D Nếu bạn có thể trình bày một chút ý tưởng về bài này thì mình nghĩ sẽ hoàn hảo :) Thực sự mới nhìn mình hơi bị quay cuồng trong phương trình 1 =))

--------------------------

Rồi, rất cảm ơn mọi người đã tham gia topic, dưới đây sẽ là một số bài " cây nhà lá vườn " :)

Bài 24. Giải hệ phương trình


$$\left\{\begin{matrix}
\sqrt{3x+y}+\sqrt{x+y}=2 & & \\
\sqrt{x+y}+x-y=1& &
\end{matrix}\right.$$

Đề thi HSG tỉnh Quảng Ninh 2011 - 2012 Bảng A

Bài 25. Giải hệ phương trình

$$\left\{\begin{matrix}
(x+1)^3-3(x+1)^2=y^3-3y^2 & & \\ x^2+\sqrt{1-x^2}-3\sqrt{2y-y^2}+2=0
& &
\end{matrix}\right.$$

Đề thi HSG Quảng Ninh 2011 - 2012 Bảng B

Nhận xét : 2 bài trên cơ bản ý tưởng cũ rồi :) Mọi người chém nhiệt tình nhoé :D



Bài 24:

Đặt: $\left\{\begin{matrix}
\sqrt{3x+y}=u\\
\sqrt{x+y}=v
\end{matrix}\right.$

với $u,v \ge 0$

Khi đó ta có:

$\left\{\begin{matrix}
u+v=2\\
v+u^2-2v^2=1
\end{matrix}\right.$

$\Leftrightarrow \left\{\begin{matrix}
u=2-v\\
(2-v)^2-2v^2+v=1
\end{matrix}\right.$

$\Leftrightarrow \left\{\begin{matrix}
u=\frac{7-\sqrt{21}}{2}\\
v=\frac{-3+\sqrt{21}}{2}
\end{matrix}\right.$

Từ đây dễ dàng tính được: $\left\{\begin{matrix}
x=5-\sqrt{21}\\
y=\frac{5-\sqrt{21}}{2}
\end{matrix}\right.$

Bài 25: (Không những cũ mà còn lộ :D )

$\left\{\begin{matrix}
(x+1)^3-3(x+1)^2=y^3-3y^2 & & \\ x^2+\sqrt{1-x^2}-3\sqrt{2y-y^2}+2=0
& &
\end{matrix}\right.$

Xét hàm số $f(t)=t^3-3t^2$ với $t \in [0;2]$ dễ thấy f(t) nghịch biến trên đoạn ta xét.

Do đó $(x+1)^3-3(x+1)^2=y^3-3y^2 $

$\Leftrightarrow x+1=y$

Thế vào pt 2 ta được:

$x^2+\sqrt{1-x^2}-3\sqrt{2y-y^2}+2=0$

$\Leftrightarrow x^2+\sqrt{1-x^2}-3\sqrt{1-x^2}=0$

$\Leftrightarrow x^2=2\sqrt{1-x^2}$

$\Leftrightarrow x=\pm \sqrt{-2+2\sqrt{2}}$

$\Rightarrow y=\pm \sqrt{-2+2\sqrt{2}}+1$

KL được nghiệm $(x;y)$



#335661 GPT : $\frac{x^{2}}{(1+\sqrt{1+x...

Đã gửi bởi kainguyen on 14-07-2012 - 17:28 trong Đại số

Giải phương trình
$\frac{x^{2}}{(1+\sqrt{1+x})^{2}}=x-4$



Ta có:

$\frac{x^{2}}{(1+\sqrt{1+x})^{2}}=x-4$

$\Leftrightarrow \frac{x^2(1-\sqrt{1+x})^2}{x^2}=x-4$

$\Leftrightarrow 1+x+1-2\sqrt{x+1}=x-4$

$\Leftrightarrow \sqrt{x+1}=3$

$\Leftrightarrow x=8$



#335431 Chứng minh: \[\left( {1 + {a^2}} \right)\left( {1 + {b^2}...

Đã gửi bởi kainguyen on 13-07-2012 - 23:06 trong Bất đẳng thức và cực trị

Cho a,b,c là các số thực không âm có tổng bằng 1 . CMR.
\[\left( {1 + {a^2}} \right)\left( {1 + {b^2}} \right)\left( {1 + {c^2}} \right) \ge {\left( {\frac{{10}}{9}} \right)^3}\]
------

@ WWW:
1. Học gõ $\LaTeX$
tại đây.
2. Xem cách đặt tiêu đề cho bài viết
tại đây.
3. Bạn cho mình hỏi đây là bài trong mục nào của THTT và số nào (tháng nào, năm nào) vậy?


Bài Đề ra kì này, số tháng 6/2012.



#335362 CMR: $c+d>a+b$

Đã gửi bởi kainguyen on 13-07-2012 - 21:23 trong Bất đẳng thức và cực trị

Cho $a$, $b$, $c$ là các số thực dương. Chứng minh rằng trong các bất đẳng thức sau có ít nhất một bất đẳng thức sai:
$c+d>a+b$, $ab>(a+b)(c+d)$, $ab(c+d)>(a+b)cd$


Giả sử $c+d>a+b$ sai. Ta có đpcm

Giả sử $c+d>a+b$ đúng.

TH1: $ab>(a+b)(c+d)$ sai ta có đpcm.

TH2: $ab>(a+b)(c+d)$ đúng, ta có: $ab>(a+b)(c+d)>(a+b)^2$ (do $c+d>a+b$)

Suy ra: $a^2+b^2<-ab$ vô lý do $-ab<0$

Vậy trong 3 bđt trên có ít nhất 1 bđt sai.



#335011 Giải hệ phương trình: $\left\{\begin{matrix} \frac{3...

Đã gửi bởi kainguyen on 12-07-2012 - 22:05 trong Phương trình - hệ phương trình - bất phương trình

Giải hệ phương trình :
$\left\{\begin{matrix}
\frac{3x-y}{x-3y}=x^{2}\\ \frac{3y-z}{y-3z}=y^{2}
\\\frac{3z-x}{z-3x} =z^{2}

\end{matrix}\right.$



Điều kiện: $x;y;z \ne 0$

Ta có:

$\left\{\begin{matrix}

\frac{3x-y}{x-3y}=x^{2}\\ \frac{3y-z}{y-3z}=y^{2}
\\\frac{3z-x}{z-3x} =z^{2}
\end{matrix}\right.$

$\Leftrightarrow \left\{\begin{matrix}
x=\frac{z^3-3z}{3z^2-1}\\
y=\frac{x^3-3x}{3x^2-1}\\
z=\frac{y^3-3y}{3y^2-1}
\end{matrix}\right.$

Đặt $z=tana$ với $a \in (-\frac{\pi}{2};\frac{\pi}{2})\setminus \left \{ 0 \right \}$

Từ trên suy ra: $\left\{\begin{matrix}
x=tan3a\\
y=tan9a\\
z=tan27a
\end{matrix}\right.$

$\Rightarrow tana=tan27a$

$\Leftrightarrow 27a=a+k\pi $

$\Leftrightarrow a=k\frac{\pi}{26}$ với $k$ nguyên.

Đối chiếu với điều kiện: $a \in (-\frac{\pi}{2};\frac{\pi}{2})\setminus \left \{ 0 \right \}$ suy ra $k \in \left \{\mathbb{Z}|-13<k<13;k \ne 0\right \}$

Từ đây suy ra phương trình có nghiệm:

$(x;y;z)=(k\frac{\pi}{26};k\frac{3\pi}{26};k\frac{9\pi}{26})$ và các hoán vị với $k \in \left \{\mathbb{Z}|-13<k<13;k \ne 0\right \}$